Hello, Dave~
About PT52 Sec3 #7, I understand why answer choice (a) is the right one, but I just feel like this one is almost sufficient assumption. While solving this one, I handled it with conditional logic, as you did, and it seems so sufficient. If you add 'speech->sound' link to the stimulus, you get the conclusion, right? Is it because sometimes necessary assumptions can be sufficient ones, or the other way around, as well?
Please help! Thanks always:)
LSAT Kung Fu Forum / PT52 Sec3 #7
PT52 Sec3 #7
Oct 19 2015
#1
PT52 Sec3 #7